0 of 32 Questions completed
Questions:
You have already completed the quiz before. Hence you can not start it again.
Quiz is loading…
You must sign in or sign up to start the quiz.
You must first complete the following:
0 of 32 Questions answered correctly
Your time:
Time has elapsed
You have reached 0 of 0 point(s), (0)
Earned Point(s): 0 of 0, (0)
0 Essay(s) Pending (Possible Point(s): 0)
Average score |
|
Your score |
|
In patients with non-valvular atrial fibrillation with prior stroke, TIA or CHA2DS2-VASC score of more than 2, the recommended oral anticoagulants include all of the following except?
Which of the following is not part of the recommended Mediterranean diet?
A 56 year old man comes in to your hypertension clinic for a review. He is an ex-smoker of 40 pack years, hyperlipidemia, hypertension but no history of diabetes. His BMI is 42.
What should his initial target systolic blood pressure be to help reduce his risk of having a major cardiovascular event in the future?
Flecainide is a class Ic antiarrhythmic agent used to prevent and treat tachyarrhythmias. However the drug has a narrow therapeutic index.
Which of the following would not be a compatible sign of cardiac toxicity related to this drug?
What is the most common symptom of mitral stenosis?
What is the most likely diagnosis?
What is the underlying rhythm?
Which of the following medication is contraindicated in systolic heart failure?
What is the mechanism of action of Tirofiban?
What is the most common cause of sudden cardiac death in young competitive athletes?
A 64-year-old man presents to hospital seven hours after onset of severe chest pain. His ECG shows 4-5 mm ST elevation in leads II, III and aVF, and complete heart block with a ventricular rate of 50/minute. His blood pressure is 115/60 mmHg which falls transiently to 90/50 mmHg with the administration of streptokinase, before returning to 120/55 mmHg. Four hours later, he is reviewed in the coronary care unit. His heart rate is 50/minute and his blood pressure is 85/60 mmHg. His jugular venous pressure is 4 cm with cannon waves. A third sound is audible but there are no murmurs. Breath sounds are normal. The patient complains of mild chest pain which is relieved by sitting up. The ECG remains unchanged. Urine output has declined to 5 mL/h for the last two hours
What is the most appropriate next step in management?
What is the most common cause of mitral stenosis?
About half of all patients with Hypertrophic Cardiomyopathy (HCM) have a positive family history compatible with autosomal dominant transmission.
What is the most common mutation?
What is the most important predisposing factor for acute aortic dissection in the elderly population?
With regards to valvular heart disease, which of the following cases is valve surgery most appropriate?
Which of the following is the strongest indication for an ICD placement in a patient with HOCM?
What is the most appropriate timing for non-cardiac surgery following PCI with bare metal stent?
Which of the following drugs have not been shown to improve survival in patients with heart failure with reduced ejection fraction (HFrEF)?
What is the most likely diagnosis based on the ECG below?
Which of the following drugs causes gynaecomastia?
What is the mechanism of action of eplerenone?
The following are true regarding Digoxin except?
The following drugs should be avoided in patients with congestive heart failure, except?
What is the most characteristic feature of Hypertrophic Cardiomyopathy on ECHO?
What is the mechanism of action of Abciximab?
In apparently healthy persons without hyperlipidemia but with elevated high-sensitivity C-reactive protein levels,which medication significantly reduced the incidence of major cardiovascular events?
A 68 year old man has a background of atrial fibrillation, hypertension, diabetes and chronic renal failure with a baseline serum creatinine of 210 umol/l. His current medications include cilazapril, glipizide and aspirin.
He enjoys drinking a glass of wine each night after dinner. His BP is 150/80 and cardiovascular examination is unremarkable. His friend had told him that warfarin is helpful in preventing stroke in patients with chronic AF so now he asks you if he should be on warfarin. Which of the following patient factors would make you decide against the use of warfarin in this man?
In patients with severe aortic stenosis who are not candidates for surgical replacement of the aortic valve, which of the following treatment significantly reduces mortality?
What is the most likely diagnosis based on the ECG below?
A 50 year old woman presents with acute onset left sided chest pain radiating to her neck and down her left arm. Her ECG shows 2mm ST elevation in V2-V4.
An urgent angiography subsequently shows a spontaneous coronary artery dissection involving her left anterior descending artery.Â
Which of the following intervention is least likely to be helpful?
You are treating a COPD patient for hypertension. The patient is 55 years old and is already on a calcium channel blocker. He continues to smoke tobacco. Which of the following anti-hypertensive should you avoid?
Apart from smoking, which of the following risk factor has the highest odds ratio for myocardial infarction?